Last visit was: 25 Apr 2024, 00:38 It is currently 25 Apr 2024, 00:38

Close
GMAT Club Daily Prep
Thank you for using the timer - this advanced tool can estimate your performance and suggest more practice questions. We have subscribed you to Daily Prep Questions via email.

Customized
for You

we will pick new questions that match your level based on your Timer History

Track
Your Progress

every week, we’ll send you an estimated GMAT score based on your performance

Practice
Pays

we will pick new questions that match your level based on your Timer History
Not interested in getting valuable practice questions and articles delivered to your email? No problem, unsubscribe here.
Close
Request Expert Reply
Confirm Cancel
User avatar
Intern
Intern
Joined: 21 Oct 2008
Posts: 4
Own Kudos [?]: 5 [4]
Given Kudos: 0
Send PM
User avatar
Director
Director
Joined: 17 Jun 2008
Posts: 614
Own Kudos [?]: 444 [0]
Given Kudos: 0
Send PM
User avatar
Manager
Manager
Joined: 29 Jun 2009
Affiliations: CFA Level 2 Candidate
Posts: 117
Own Kudos [?]: 1262 [0]
Given Kudos: 2
Concentration: Finance
Schools:RD 2: Darden Class of 2012
 Q49  V35
Send PM
avatar
CIO
CIO
Joined: 02 Oct 2007
Posts: 880
Own Kudos [?]: 1357 [3]
Given Kudos: 334
Re: M03 Q33 [#permalink]
3
Kudos
2, 4, 6 would be consecutive even integers; 4, 8, 12 would be consecutive multiples of 4. Consecutive integers are 1, 2, 3, 4 and so on unless otherwise stated.
hogann wrote:
Is this the correct definition of consecutive integers?

To me consecutive integers could be 1,2,3 but also 2,4,6 or 4,8,12
Is this called something else? Consecutive series?
avatar
Intern
Intern
Joined: 12 Sep 2009
Posts: 10
Own Kudos [?]: 17 [1]
Given Kudos: 2
Send PM
Re: M03 Q33 [#permalink]
1
Kudos
Nice one. Thanks for clarifying the term “consecutive XXXX”
User avatar
Manager
Manager
Joined: 02 Nov 2009
Posts: 86
Own Kudos [?]: 64 [1]
Given Kudos: 7
Location: VA
Schools:Columbia Business School '13
 Q28  V36 GMAT 2: 650  Q40  V39 GMAT 3: 720  Q44  V45
GPA: 3.6
WE 1: Investment Banking
WE 2: Higher Education
Send PM
Re: M03 Q33 [#permalink]
1
Kudos
x, y and z are integers. Are they consecutive?

1. y equals to the arithmetic mean of x and z
2. x = -z

So is the correct answer E?

Taking statement 1, y = (x+z)/2 Not sufficient.

Taking statement 2, if x is 1, then the x, y, z could be -1, 0, 1. However, they could also be -2, 0, 2. Not sufficient.

Combining the statements and substituting, y = (-z+z)/2, which simplifies to y = 0/2 or 0. If y is 0 and x = -z, we still do not have enough information to answer the question.
avatar
CIO
CIO
Joined: 02 Oct 2007
Posts: 880
Own Kudos [?]: 1357 [0]
Given Kudos: 334
Re: M03 Q33 [#permalink]
You're absolutely right. The OA is E.

phillypointgod21 wrote:
x, y and z are integers. Are they consecutive?

1. y equals to the arithmetic mean of x and z
2. x = -z

So is the correct answer E?

Taking statement 1, y = (x+z)/2 Not sufficient.

Taking statement 2, if x is 1, then the x, y, z could be -1, 0, 1. However, they could also be -2, 0, 2. Not sufficient.

Combining the statements and substituting, y = (-z+z)/2, which simplifies to y = 0/2 or 0. If y is 0 and x = -z, we still do not have enough information to answer the question.
User avatar
Senior Manager
Senior Manager
Joined: 21 Dec 2009
Posts: 337
Own Kudos [?]: 2443 [8]
Given Kudos: 20
Concentration: Entrepreneurship, Finance
Send PM
Re: M03 Q33 [#permalink]
7
Kudos
1
Bookmarks
(1) y = average of x and z:
x(1), y(2), z(3) --> y is avrg of x & z and x,y,z are consecutives
x(2), y(2), z(2) --> y is avrg of x & z but x,y,z are not consecutives
INSUFFICIENT

(1) x= -z says nothing about y...
INSUFFICIENT

(1) and (2):
x(-1), y(0), z(1)... y is avrg of x and z and x,y,z are consecutives
it might be tempting to pick C without trying another set of nos.
x(-2), y(0), z(2)... y is avrg of x and z but x,y,z are not consecutives
INSUFFICIENT

So, E.
User avatar
Senior Manager
Senior Manager
Joined: 24 Jun 2010
Status:Time to step up the tempo
Posts: 273
Own Kudos [?]: 673 [2]
Given Kudos: 50
Location: Milky way
Concentration: International Business, Marketing
Schools:ISB, Tepper - CMU, Chicago Booth, LSB
Send PM
Re: M03 Q33 [#permalink]
2
Kudos
Statement 1: y = (x+z)/2. y is the middle term but we do not know whether the difference between x, y and/or y,z is 1.
Insufficient.

Statement 2: x = -z. This tells us that y = 0, however we do not know whether about the difference between x,y and y,z.

Insufficient.

Combining both: we know that y is the middle term and the mean of x and z but still we cannot determine that the difference between x,y and y,z is 1 (condition for x, y, z to be consecutive.)

Answer E.
User avatar
Intern
Intern
Joined: 01 Oct 2010
Posts: 2
Own Kudos [?]: 12 [0]
Given Kudos: 1
Send PM
Re: M03 Q33 [#permalink]
Agree with E for sure.

State 1: y = A.M of X & Y

We get inconclusive answers as if X=3 , Y=4, Z=5
Then Y=(3+5)/2=4 = A.M is true ans consecutive no. but say if X=2, Y=5, Z=8
then also Y=A.M =5 but the number are no consecutive so insufiecient

Answers could be B, C or e

Stat II: X=-Z gives us nothing so insuffiecient .

Taking them together:

If z=2 then X=-2 then y=0 not consecutive
Watch out here as all values except 1 the no will come out to be not consecutive.
Except 1 If z=1 then x=-1 Y=0 consecutive no. so we have conflicting answers so correct Answer is [E]

Hope this helps
User avatar
Intern
Intern
Joined: 16 May 2010
Status:Current Student
Posts: 38
Own Kudos [?]: 13 [0]
Given Kudos: 4
Concentration: GMO/Sustainability
Schools:<strong>Darden '13</strong>
Send PM
Re: M03 Q33 [#permalink]
gmatbull wrote:
(1) y = average of x and z:
x(1), y(2), z(3) --> y is avrg of x & z and x,y,z are consecutives
x(2), y(2), z(2) --> y is avrg of x & z but x,y,z are not consecutives
INSUFFICIENT

(1) x= -z says nothing about y...
INSUFFICIENT

(1) and (2):
x(-1), y(0), z(1)... y is avrg of x and z and x,y,z are consecutives
it might be tempting to pick C without trying another set of nos.
x(-2), y(0), z(2)... y is avrg of x and z but x,y,z are not consecutives
INSUFFICIENT

So, E.


Thanks. To me, you did the best job of explaining why the answer isn't "A". Just because all consecutive integers will work for X+Z/2=Y doesn't mean that ONLY consecutive integers will do that.
avatar
Manager
Manager
Joined: 20 Nov 2010
Posts: 88
Own Kudos [?]: 117 [1]
Given Kudos: 38
Send PM
Re: M03 Q33 [#permalink]
1
Kudos
Its E.
St1: Every AP has the same property b = (a+c)/2 . So insufficient.
St2: There are infinite series with a = -c eg -5,0,5 ; -2,0,2 So insufficient.

Both statements together doesn't narrow it down. Hence E.
User avatar
Manager
Manager
Joined: 06 Feb 2011
Affiliations: University of Tehran
Posts: 170
Own Kudos [?]: 46 [0]
Given Kudos: 57
Location: Iran (Islamic Republic of)
Grad GPA: 4
Concentration: Marketing
Schools: Wharton
GMAT 1: 680 Q45 V38
GPA: 4
WE:Marketing (Retail)
Send PM
Re: M03 Q33 [#permalink]
Great question. Made me make a mistake. I simply took Z=1 and X=-1 according to stmt 2. Then according to stmt 1, y must be 0 here. So the three numbers comprise a consecutive integer -1,0,1. But haha what about -2, 0 , 2 which is supported by the statements? It's no a consecutive integer series. So E
User avatar
Manager
Manager
Joined: 24 Jun 2011
Status:MBAing!!!!
Posts: 167
Own Kudos [?]: 70 [0]
Given Kudos: 56
Location: United States (FL)
Concentration: Finance, Real Estate
GPA: 3.65
WE:Project Management (Real Estate)
Send PM
Re: M03 Q33 [#permalink]
I picked E

Stm.1- y=(x+z)/2
2y=x+z
3y=x+y+z
y=(x+y+z)/3 ...this statement only means that the mean=median... consecutive and nonconsecutive integers satisfy this condition...therefore not Sufficient.

Stm.2- No data about y...therefore insufficient.

Stm1 and stm2 together (x,y,z)=(-3,0,3) or (-1,0,1) therefore not sufficient.
Math Expert
Joined: 02 Sep 2009
Posts: 92901
Own Kudos [?]: 618870 [1]
Given Kudos: 81588
Send PM
Re: M03 Q33 [#permalink]
1
Kudos
Expert Reply
REVISED VERSION OF THIS QUESTION IS BELOW:

Are integers \(x\), \(y\), and \(z\) consecutive?

(1) The average (arithmetic mean) of \(x\) and \(z\) equals to \(y\) --> \(y=\frac{x+z}{2}\): if \(x=y=z=0\) then the answer is NO but if \(x=-1\), \(y=0\) and \(z=1\) then the answer is YES. Not sufficient.

(2) \(x= -z\). Not sufficient since no info about \(y\).

(1)+(2) Examples discussed for statement (1) are still valid, so we can have a NO (\(x=y=z=0\)) as well as an YES (\(x=-1\), \(y=0\) and \(z=1\)) answers. Not sufficient.

Answer: E.
Senior Manager
Senior Manager
Joined: 17 Mar 2010
Status:Final Countdown
Posts: 320
Own Kudos [?]: 1305 [0]
Given Kudos: 76
Location: United States (NY)
GPA: 3.82
WE:Account Management (Retail Banking)
Send PM
Re: M03 Q33 [#permalink]
Are integers \(x\), \(y\), and \(z\) consecutive?

1. \(y\) equals the arithmetic mean of \(x\) and \(z\)
2. \(x = -z\)

Nice workout !

(i) take 5,6,7
6 is the AM of 5&7 yes Consecutive
take 10,12,14
12 is the AM of 10&14 Not consecutive

insufficient

(ii)take -1,0,1 as x,y & z- possible because x=(-z)
but if we take z=-5, x will be 5 , then it will not be consecutive-not possible

insufficient

Answer-E

Archived Topic
Hi there,
Archived GMAT Club Tests question - no more replies possible.
Where to now? Try our up-to-date Free Adaptive GMAT Club Tests for the latest questions.
Still interested? Check out the "Best Topics" block above for better discussion and related questions.
Thank you for understanding, and happy exploring!
GMAT Club Bot
Re: M03 Q33 [#permalink]
Moderator:
Math Expert
92901 posts

Powered by phpBB © phpBB Group | Emoji artwork provided by EmojiOne